Problem with calculating the cov matrix of X,Y

  • Thread starter ChrisVer
  • Start date
  • Tags
    Matrix
In summary, if I have two random variables X, Y that are given from the following formula: X= \mu_x \big(1 + G_1(0, \sigma_1) + G_2(0, \sigma_2) \big) Y= \mu_y \big(1 + G_3(0, \sigma_1) + G_2(0, \sigma_2) \big)then to find the covariance matrix of those two, I would first need to find the variance and covariance of each variable. Then, I could use those values to find the covariance matrix.
  • #1
ChrisVer
Gold Member
3,378
464
If I have two random variables [itex]X, Y[/itex] that are given from the following formula:
[itex] X= \mu_x \big(1 + G_1(0, \sigma_1) + G_2(0, \sigma_2) \big) [/itex]
[itex] Y= \mu_y \big(1 + G_3(0, \sigma_1) + G_2(0, \sigma_2) \big)[/itex]

Where [itex]G(\mu, \sigma)[/itex] are gaussians with mean [itex]\mu=0[/itex] here and std some number.

How can I find the covariance matrix of those two?

I guess the variance will be given by:
[itex]Var(X) = \mu_x^2 (\sigma_1^2+ \sigma_2^2)[/itex] and similarly for Y. But I don't know how I can work to find the covariance?
Could I define some other variable as :[itex]Z=X+Y[/itex] and find the covariance from [itex]Var(Z)= Var(X)+Var(Y) +2 Cov(X,Y)[/itex] ?
while [itex]Z[/itex] will be given by [itex]Z= (\mu_x+\mu_y) (1+ G_1 + G_2) [/itex]?

Then [itex]Var(Z)= (\mu_x+ \mu_y)^2 (\sigma_1^2+ \sigma_2^2)[/itex]

And [itex]Cov(X,Y) = \dfrac{(\mu_x+\mu_y)^2(\sigma_1^2+ \sigma_2^2)- \mu_x^2 (\sigma_1^2+ \sigma_2^2) - \mu_y^2(\sigma_1^2+ \sigma_2^2) }{2}=\mu_x \mu_y(\sigma_1^2+ \sigma_2^2) [/itex]

Is my logic correct? I'm not sure about the Z and whether it's given by that formula.
 
Last edited:
Physics news on Phys.org
  • #2
Why not simply apply the definition of covariance? Or the reduced form E(XY)-E(X)E(Y)?
 
  • #3
I don't know E[XY]...?
 
  • #4
But you know the distributions of ##X## and ##Y## and so you can compute it.
 
  • #5
I don't know the joint distribution function...
so that [itex]E[xy]= \int dxdy~ h(x,y) xy[/itex]
And I can't say [itex]h(x,y)=f(x)g(y)[/itex] since I don't know if X,Y are independent...if they were independent they wouldn't have a covariance too...the E[xy]=E[x]E[y] and your reduced form formula would vanish the cov.
 
  • #6
ChrisVer said:
I don't know the joint distribution function...

You do. The only reason I can see to number the ##G##s is to underline out that they are independent distributions so that ##X## and ##Y## have a common part ##G_2## and one individual part ##G_1##/##G_3##.
 
  • #7
Yes that's the reason of labeling them... It just happens that ##G_1,G_2## have the same arguments ##\mu=0, \sigma_1## but they are not common for X,Y. However the [itex]G_3[/itex] is a common source of uncertainty in both X,Y... Still I don't understand how to get the joint probability from it... like taking the intersection of X,Y is leading to only the ##G_3##? that doesn't seem right either.
 
  • #8
I suggest you start from the three-dimensional distribution for ##G_i##. From there you can integrate over regions of constant ##X## and ##Y## to obtain the joint pdf for ##X## and ##Y##. In reality, it does not even have to be that hard. Just consider ##X## and ##Y## as functions on the three dimensional outcome space the ##G_i## and use what you know about those, e.g., ##E(G_1G_2) = 0## etc.
 
  • #9
ChrisVer said:
If I have two random variables [itex]X, Y[/itex] that are given from the following formula:
[itex] X= \mu_x \big(1 + G_1(0, \sigma_1) + G_2(0, \sigma_2) \big) [/itex]
[itex] Y= \mu_y \big(1 + G_3(0, \sigma_1) + G_2(0, \sigma_2) \big)[/itex]

To keep the discussion clear, you should use correct notation. If "[itex] X [/itex]" is a random variable then [itex] X [/itex] has a distribution, but it is not "equal" to its distribution. I think what you mean is that:

[itex] X = \mu_x( X_1 + X_2) [/itex]
[itex] Y = \mu_y( X_3 + X_2 )[/itex]

where [itex] X_i [/itex] is a random variable with Gaussian distribution [itex] G(0,\sigma_i) [/itex].
 
  • Like
Likes ChrisVer
  • #10
No, X is a random variable,taken from a distribution function with:
mean ##\mu_x## (that's the role of 1)
some measurment uncertainty following a guassian ##G_1## or ##G_3##.
a further common measurement uncertainty ##G_3##.

That means I could take 5 measurements from X :##\{x_1,x_2,x_3,x_4,x_5 \}=\mu_x+ \{ + 0.02, + 0.001, 0, - 0.01, - 0.06\}##.
 
  • #11
Ahhh OK I understood what you meant to say... yes it's fine, I understand what I wrote maybe I didn't write it in a correct way (confusing random variables with distributions)...
 
  • #12
Apply the formula for [itex]\sigma(aX + bY, cW + dV) [/itex] given in the Wikipedia article on Covariance http://en.wikipedia.org/wiki/Covariance

In your case , [itex] a = b = \mu_x,\ X=X_1,\ Y= X_2,\ c=d=\mu_y,\ W = X_3, V = X_2 [/itex]

Edit: You'll have to put the constant 1 in somewhere. You could use X = 1 + X_1.
 
  • #13
I also thought about this:
writing the covariance as [itex] \sigma_{XY} = \sigma_X \sigma_Y \rho[/itex] with [itex]\rho[/itex] the correlation coefficient.

And since [itex]X= \mu_x (1+ X_1 + X_2)[/itex] and [itex]Y = \mu_y (1 + X_3 + X_2) [/itex] I think these two are linearly correlated (due to [itex]X_2[/itex]). So [itex]\rho>0[/itex]. Would you find this a logical statement? I mean if [itex]X_2[/itex] happens to be chosen a larger number, both [itex]X,Y[/itex] will get a larger number as contribution.
For the value of [itex]\rho[/itex] I guess it should (by the same logic) be given by a combination of [itex]\mu_y,\mu_x[/itex], since they give the difference in how [itex]X,Y[/itex] change with [itex]X_2[/itex]'s change. I mean if [itex]\mu_x > \mu_y[/itex] then [itex]X[/itex] will get a larger contribution from the same [itex]X_2[/itex] than [itex]Y[/itex] and vice versa for [itex]\mu_x<\mu_y[/itex]...So I guess it should be an [itex]X(X_2)=\frac{\mu_x}{\mu_y} Y(X_2)[/itex]?
 
Last edited:
  • #14
I still think you are overcomplicating things. Do you know how to compute ##E(X_iX_j)## when ##X_i## has a Gaussian distribution with mean zero?
 
  • #15
In general I'd know, but as I said I have difficulty in finding the joint pdf...

Are you saying that [itex]\mu_i (1+G_2)[/itex] in my notation is the joint pdf (after integrating out G1 and G3)?
 
  • #16
You do not need to find the joint pdf. You can work directly with the Gaussians!
 
  • #17
Then in that case I don't know how to find ##E[X_i X_j]##...
The formula I know defines the expectation value through an integral with the pdf...:sorry:
 
  • #18
That one is simple, it is the expectation value of two independent gaussians with zero mean. Since they are independent, the pdf factorises ...

Edit: ... unless, of course, if i = j ...
 
  • Like
Likes ChrisVer
  • #19
So you suggest something like:
[itex]E[X_i X_j] = \begin{cases} E[X_i]E[X_j] = \mu_i \mu_j & i \ne j \\ E[X_i^2]= \sigma_i^2 + \mu_i^2 & i=j \end{cases}[/itex]

Where [itex]X_i[/itex] gaussian distributed variables
 
  • #20
Indeed.

Edit: Of course, you now have ##E[(1+X_1+X_2)(1+X_3+X_2)]## so you will have to do some algebra, but not a big deal.
 
  • Like
Likes ChrisVer
  • #21
[itex]E[XY] = \mu_x \mu_y E[ (1+G_1 + G_2) (1+G_3 + G_2) ] = \mu_x \mu_y \Big( 1 + 4E[G_i] + 3 E[G_i G_j] + E[G_2G_2] \Big)_{i\ne j} = \mu_x \mu_y (1+ \sigma_2^2 )[/itex] ?

So [itex]Cov(X,Y) = E[XY]- \mu_x \mu_y = \mu_x \mu_y \sigma_2^2[/itex].
That's different to the result I got from [itex]Z=X+Y[/itex]... would you know the reason?
 
Last edited:
  • #22
I suspect it comes from this not being a correct expression for ##Z##.
ChrisVer said:
while [itex]Z[/itex] will be given by [itex]Z= (\mu_x+\mu_y) (1+ G_1 + G_2) [/itex]?
The correct expression is
##
Z = (\mu_x + \mu_y)(1+G_2) + \mu_x G_1 + \mu_2 G_3.
##
Even though ##G_1## and ##G_3## have the same distribution, they are not the same random variable.
 
  • Like
Likes ChrisVer
  • #23
Yup that's right...
[itex]
\begin{align}

Var[Z]=& Var[M_e] + Var[M_f] + 2 Cov(M_e,M_f) \notag\\

\text{also} =& (m_e+m_f)^2 \sigma_s^2 + m_e^2 \sigma_e^2 + m_f^2 \sigma_f^2 \notag \\

&\Downarrow \notag \\

Cov(M_e,M_f) =& \dfrac{(m_e+m_f)^2 \sigma_s^2 + m_e^2 \sigma_e^2 + m_f^2 \sigma_f^2- m_e^2 (\sigma_e^2+\sigma_s^2) - m_f^2 (\sigma_f^2 + \sigma_s^2)}{2} \notag \\

=&\dfrac{2 m_e m_f \sigma_s^2}{2} = m_e m_f \sigma_s^2

\end{align}

[/itex]

So the mistake was that I got the two G's after sum to give a same G.
 
  • #24
[itex] Cov( \mu_x( 1 + X_1 + X_2), \mu_y (1 + X_3 + X_2) ) = \mu_x \mu_y Cov(1 + X_1 + X_2, 1 + X_3 + X_2) [/itex]
[itex] = \mu_x \mu_y Cov( X_1 + X_2, X_3 + X_2) [/itex]
[itex] = \mu_x \mu_y ( Cov(X_1,X_3) + Cov(X_1,X_2) + Cov(X_2,X_3) + Cov(X_2,X_2)) [/itex]
Assuming the [itex] X_i [/itex] are independent random variables
[itex] = \mu_x \mu_y ( 0 + 0 + 0 + Var(X_2) ) [/itex]
 
  • Like
Likes ChrisVer
  • #25
So far the covariance matrix is:
[itex] C[X_1,X_2]= \begin{pmatrix} m_x^2 (\sigma_1^2+ \sigma_s^2) & m_x m_y \sigma_s^2 \\ m_x m_y \sigma_s^2 & m_y^2 (\sigma_2^2 +\sigma_s^2) \end{pmatrix}[/itex]

I am bringing up a conversation I had in class about that... Since I am still unable to understand how this thing could work out.
In a further step we said that suppose that [itex]X_1,X_2[/itex] are uncorrelated, that would mean that their covariance is [itex]Cov(X_1,X_2)=0[/itex]. We would need to find what the [itex]\sigma_D=\sqrt{Var[D]}[/itex] is, where [itex]D= X_2-X_1[/itex].
Obviously [itex]Var[D]= Var[X_2] + Var[X_1] - 2Cov[X_1,X_2][/itex]

The main problem I had with this conversation is that I was told I should get [itex]Cov[X_1,X_2]=0[/itex] in the above formula. I was stating instead that in order for the covariance to be 0, I would have to send [itex]\sigma_s =0[/itex] and that would also influence the variances of X1,X2: [itex]Var[X_1]= m_x^2 ( \sigma_1^2 + \sigma_s^2) = m_x^2 \sigma_1^2[/itex].

The thing is that in my case I'm dropping out the [itex]\Delta_s[/itex] from the initial expressions: [itex]X_1 = m_x (1+ \Delta_1 + \Delta_s)[/itex] , [itex]X_2 = m_y (1+\Delta_2 + \Delta_s)[/itex], where [itex]\Delta_s \sim Gaus(0,\sigma_s)[/itex] can be seen a systematic error in the measurement of [itex]X_1,X_2[/itex] and the [itex]\Delta_{1,2}[/itex] would only be the statistical errors in the measurement.

The guy I talked about this told me it's wrong since in the X1 case the [itex]\Delta_s= \Delta_{s1}+ \Delta_{s2} +...[/itex] and in X2: [itex]\Delta_s = \Delta_{s2}[/itex] and their correlation could only come from the [itex]\Delta_{s2}[/itex] (so dropping the [itex]\Delta_s[/itex]'s was wrong because I was dropping the [itex]\Delta_{s1},\Delta_{s3},...[/itex] from X1). And [itex]\Delta_{si}[/itex] are unknown individual systematic errors coming from the measurement.

What do you think? Sorry if it sounds a little bit complicated...
 
Last edited:

1. Why is it important to calculate the covariance matrix of X and Y?

The covariance matrix of X and Y provides information about the relationship between the two variables. It allows us to understand how changes in one variable affect the other, and can help us identify patterns and trends in the data.

2. What does a positive covariance between X and Y indicate?

A positive covariance indicates that the two variables are positively related, meaning that when one variable increases, the other tends to increase as well. This suggests a direct relationship between the variables.

3. How does the size of the covariance affect the relationship between X and Y?

The size of the covariance indicates the strength of the relationship between X and Y. A larger covariance suggests a stronger relationship, while a smaller covariance suggests a weaker relationship.

4. Can the covariance matrix of X and Y be used to predict future values?

No, the covariance matrix only describes the relationship between X and Y in the current dataset. It cannot be used to make predictions about future values.

5. What is the difference between covariance and correlation?

Covariance measures the strength and direction of the relationship between two variables, while correlation measures the strength and direction of the linear relationship between two variables. Correlation is a standardized version of covariance and is always between -1 and 1, while covariance has no upper or lower limit.

Similar threads

  • Set Theory, Logic, Probability, Statistics
Replies
1
Views
770
  • Set Theory, Logic, Probability, Statistics
2
Replies
43
Views
4K
  • Calculus and Beyond Homework Help
Replies
1
Views
540
  • Set Theory, Logic, Probability, Statistics
Replies
3
Views
2K
  • Set Theory, Logic, Probability, Statistics
Replies
10
Views
1K
  • Set Theory, Logic, Probability, Statistics
Replies
1
Views
1K
  • Set Theory, Logic, Probability, Statistics
Replies
4
Views
851
  • Set Theory, Logic, Probability, Statistics
Replies
11
Views
1K
  • Set Theory, Logic, Probability, Statistics
Replies
11
Views
2K
  • Set Theory, Logic, Probability, Statistics
Replies
20
Views
3K
Back
Top